Logo Studenta

BertJanssen-RelatividadGeneral-39

¡Estudia con miles de materiales!

Vista previa del material en texto

última igualdad hemos utilizado las expresiones (1.107) y (1.108) para el gradiente. La ecuación
(1.109) es por lo tanto la ley de Gauss para cargas magnéticas: el flujo total a través de una su-
perficie cerrada es igual a la carga encerrada dentro de la superficie, en acuerdo con la solución
(1.102) propuesta al principio.
Con esta construcción la singularidad de la cuerda de Dirac no aparece en la descripción, de-
mostrando que no esmás que un artefacto de la elección de gauge. Clásicamente el procedimiento
de utilizar potenciales distintos en regiones distintas del espacio es directo y sin consecuencias.
Cuánticamente es más sutil, puesto que antes hemos visto que al cambiar de gauge, la función
de onda de un electrón adquiere una fase. Por lo tanto, las funciones de onda que describen el
electrón con carga qe en el hemisferio norte y sur están relacionadas mediante
Ψ(n)(~r) = exp
(
− iqeqm
2π~
ϕ
)
Ψ(s)(~r). (1.110)
Sin embargo, este cambio de fase tiene consecuencias profundas: obviamente en el ecuador θ =
π/2, la función de onda tiene que ser univaluada al dar una vuelta alrededor de la esfera. En otras
palabras, la fase en (1.110) ha de tener el mismo valor para ϕ = 0 y ϕ = 2π, lo que implica que
su argumento qeqm/2π~ tiene que ser un número entero n. Escrito de otra forma, llegamos a la
llamada condición de cuantización de carga de Dirac:10
qe =
2πn~
qm
. (1.111)
En otras palabras, una consecuencia directa de la existencia de monopolos magnéticos es que las
cargas eléctricas aparecen cuantizadas, en multiplos enteros de una carga mı́nima qe = 2π~/qm.
Ni siquiera es necesario que el monopolo magnético esté cerca, con que hubiera un solo monopo-
lo magnético en alguna parte del universo, todas las cargas eléctricas estarı́an cuantizadas. Esto
es un resultado francamente sorprendente, ya que a pesar de que nunca se haya visto (con se-
guridad) un monopolo magnético, la cuantización de carga es precisamente lo que ocurre en la
naturaleza: todas las partı́culas conocidas tienen una carga que es un multiplo entero de la carga
del electrón.11 Por ejemplo, aunque el electrón y el protón son dos partı́culas muy distintas, la
cota experimental de la diferencia de sus cargas es
|qe− + qp|
e
< 1, 0 · 10−21. (1.112)
Obviamente esto no puede servir como prueba de la existencia de monopolos, pero por otro lado
no se conoce ningún otro mecanismo convincente que pueda explicar la cuantización de la carga.
Es más, en 1974 el fı́sico holandés Gerard ’t Hooft (1946) y el ruso Alexander Polyakov (1945)
intentaron cuantizar la carga eléctrica por otros medios, embebiendo el electromagnetismo en
una teorı́a gauge mas grande, pero encontraron que los llamados monopolos de ’t Hooft-Polyakov
aparecen por la puerta trasera como defectos topológicos, al romper la simetrı́a para recuperar la
teorı́a de Maxwell. En otras palabras, parece que (de momento) no hay manera de cuantizar la
carga eléctrica sin de algún modo introducir monopolos magnéticos también.
Igual que en el caso del efecto Aharonov-Bohm, la base matemática del monopolo de Dirac es
una estructura topológica no-trivial. El espacio de configuraciones es R3\{0}, puesto que el po-
tencial es divergente en ~r = 0. (Hemos eliminado el resto de la cuerda de Dirac con el truco deWu
y Yang, pero la singularidad en el origen aparece en ambos potenciales). Estamos por lo tanto tra-
bajando con un fibradoU(1) sobre el espacioR3\{0}, que tiene lamisma homotopı́a que S2. La S2
10La misma formula se puede derivar también utilizando un sólo potencial en el espacio entero. Para evitar un efecto
Aharonov-Bohm visible alrededor de la cuerda de Dirac, hay que ajustar la fase tal que sea un multiplo de 2π.
11Es bien sabido que los quarks sı́ tienen una carga q = ±e/3 ó q = ±2e/3, por lo que parecen violar la condición
de cuantización de carga. Sin embargo QCD, la teorı́a gauge que describe el comportamiento de los quarks, predice un
efecto llamado libertad asintótica, responsable del hecho de que los quarks no puedan aparecer libremente. Debido a este
confinamiento de quarks, éstos se manifiestan siempre en conjuntos de dos o tres, tal que la carga total del conjunto es un
multiplo entero de la carga del electrón.
39

Continuar navegando